Etude cinématique d'un solide en mouvement de translation

De la mécanique au nucléaire, nos physiciens sont à l'écoute
ouss10
Messages: 6
Enregistré le: 06 Jan 2013, 14:07

Etude cinématique d'un solide en mouvement de translation

par ouss10 » 06 Jan 2013, 22:36

Un mobile M supposé ponctuel se déplace dans un plan muni d'un repère orthonormé (o,i,j) . A chaque instant le vecteur accélération est a = 2 . j . A la date t = 1 s , le vecteur vitesse est v = i - 3 j et le vecteur position OM = 3 . i - 4 . j ps: ( i , j , a et OM avec vecteur ) .
Determiner en fonction du temps les expressions du vecteur vitesse v et du vecteur position OM ??



Black Jack

par Black Jack » 07 Jan 2013, 18:53

vx(t) = 1
vy(t) = A + 2t (avec Vy(1) = -3) ---> -3 = A + 2 et donc A = -5
vy(t) = -5 + 2t

vecteur v(t) = 1 . vect(i) + (2t-5) . vect(j)

vecteur v(t) = d (vect(OM))/dt

vect OM(t) = S vecteur v(t) dt (avec S pour le signe intégrale).

vect OM(t) = (S dt) * vect i + (S (2t-5) dt) * vect j

... ne pas oublier les constantes d'intégration, dont on pourra calculer les valeurs numériques en tenant compte des conditions initiales vect OM(1) = 3 * vect i - 4 * vect j

Il me semble qu'on arrive finalement à :

vect OM(t) = (t + 2) . vect(i) + (t² - 5t) . vect(j)
************
Il reste à compléter les morceaux qui manquent ... et vérifier si je me suis planté ou non.

:zen:

ouss10
Messages: 6
Enregistré le: 06 Jan 2013, 14:07

par ouss10 » 07 Jan 2013, 22:49

bon voila mon travail :
j'ai fait le primitive du vecteur a
donc j'ai V= cte1 . i + (2t + cte2 ) . j
or a t=1 on a V1= i - 3 . j
donc cte1= 1 et cte2 = 1
d'ou vecteur vitesse V= i + ( 2t +1 ) . j
est ce que c la bonne reponse ?

Black Jack

par Black Jack » 09 Jan 2013, 15:25

ouss10 a écrit:bon voila mon travail :
j'ai fait le primitive du vecteur a
donc j'ai V= cte1 . i + (2t + cte2 ) . j
or a t=1 on a V1= i - 3 . j
donc cte1= 1 et cte2 = 1
d'ou vecteur vitesse V= i + ( 2t +1 ) . j
est ce que c la bonne reponse ?


L'énoncé précise "A la date t = 1 s , le vecteur vitesse est v = i - 3 j"

et ta réponse : V= i + ( 2t +1 ) . j pour t = 1 est v = i + 3j

Ce n'est donc pas correct.

:zen:

 

Retourner vers ⚛ Physique

Qui est en ligne

Utilisateurs parcourant ce forum : Aucun utilisateur enregistré et 8 invités

Tu pars déja ?



Fais toi aider gratuitement sur Maths-forum !

Créé un compte en 1 minute et pose ta question dans le forum ;-)
Inscription gratuite

Identification

Pas encore inscrit ?

Ou identifiez-vous :

Inscription gratuite